Extending a field by a 16th primitive root of unity

Click For Summary
The discussion centers on identifying subfields M of Q(c), where c is a primitive 16th root of unity, such that the degree of the extension [M:Q] equals 2. Initially, Q(c^8) was proposed as the only candidate, but it was pointed out that Q(c^8) is actually equal to Q, resulting in a degree of 1 instead of 2. The conversation highlights a misunderstanding regarding the relationship between the degree of the extension and the order of the element in the multiplicative group. It clarifies that Q(c^4) and Q(c^12) both have a degree of 4, not 2, and emphasizes the need to correctly assess the degrees of these extensions. The conclusion is that no subfield M exists with [M:Q] = 2 in this context.
PsychonautQQ
Messages
781
Reaction score
10

Homework Statement


let c be a primitive 16th root of unity. How many subfields M<Q(c) are there such that [M:Q] = 2

Homework Equations

The Attempt at a Solution


I think the only subfield M of Q(c) such that [M:Q] = 2 is Q(c^8). Then M = {a+b(c^8) such that a,b are elements of Q}. I'm thinking about the other powers of c and trying to think if any other would generate an extension field over Q with a degree of 2. Any number that's relatively prime to 16 would be another primitive 16th root of unity, so we can throw out all odd numbers. Q(c^2) and Q(c^14) would both be degree 8, Q(c^4) and Q(c^12) would both be degree four, Q(c^6) would be of degree 8 and Q(c^10) would also be of degree 8. So it's only [Q(c^8):Q]= 2 correct?
 
Physics news on Phys.org
PsychonautQQ said:
I think the only subfield M of Q(c) such that [M:Q] = 2 is Q(c^8).
Think again about that '8' exponent. Note that ##c^8=-1##, which is in ##\mathbb Q##, so ##\mathbb Q(c^8)=\mathbb Q## (ie the extension is trivial), whence ##[\mathbb Q(c^8):\mathbb Q]=1##, not 2.
PsychonautQQ said:
Q(c^4) and Q(c^12) would both be degree four
Why? I think you are confusing the degree of the extension ##[\mathbb Q(c^k):\mathbb Q]##, which is the dimension of the vector space, with the order of the element ##c^k## in the multiplicative group ##\{c^j\ :\ j\in\{0,1,...,15\}\}##.
 
Question: A clock's minute hand has length 4 and its hour hand has length 3. What is the distance between the tips at the moment when it is increasing most rapidly?(Putnam Exam Question) Answer: Making assumption that both the hands moves at constant angular velocities, the answer is ## \sqrt{7} .## But don't you think this assumption is somewhat doubtful and wrong?

Similar threads

  • · Replies 4 ·
Replies
4
Views
4K
  • · Replies 3 ·
Replies
3
Views
2K
  • · Replies 3 ·
Replies
3
Views
2K
  • · Replies 11 ·
Replies
11
Views
2K
  • · Replies 1 ·
Replies
1
Views
1K
  • · Replies 14 ·
Replies
14
Views
2K
Replies
13
Views
2K
Replies
1
Views
2K
  • · Replies 1 ·
Replies
1
Views
2K
  • · Replies 4 ·
Replies
4
Views
2K